The 3 (Not) Real Values

Algebra Level 4

{ a + b + c = 7 a 2 + b 2 + c 2 = 23 1 a + b + 1 b + c + 1 a + c = 31 \large \begin{cases} {a+b+c=7} \\ {a^2+b^2+c^2= 23} \\ {\frac1{a+b} +\frac1{b+c} + \frac1{a+c} = 31 } \end{cases}

If a , b a,b and c c satisfy the system of equations above, find the value of a 3 + b 3 + c 3 a^3 + b^3 + c^3 .


The answer is 337.

This section requires Javascript.
You are seeing this because something didn't load right. We suggest you, (a) try refreshing the page, (b) enabling javascript if it is disabled on your browser and, finally, (c) loading the non-javascript version of this page . We're sorry about the hassle.

18 solutions

Hs N
Jan 28, 2014

This can easily be solved backwards: first we manipulate the last equation a bit until it reads 31 = a 2 + b 2 + c 2 + 3 ( a b + b c + a c ) ( a + b ) ( b + c ) ( a + c ) = 23 + 3 ( ( a + b + c ) 2 ( a 2 + b 2 + c 2 ) 2 ) ( a + b ) ( b + c ) ( a + c ) = 23 + 3 2 ( 49 23 ) ( a + b ) ( b + c ) ( a + c ) , 31=\frac{a^2+b^2+c^2+3(ab+bc+ac)}{(a+b)(b+c)(a+c)} = \frac{23+3\large{(}\frac{(a+b+c)^2-(a^2+b^2+c^2)}{2}\large{)}}{(a+b)(b+c)(a+c)}=\frac{23+\frac{3}{2}(49-23)}{(a+b)(b+c)(a+c)}, which equals 62 ( a + b ) ( b + c ) ( a + c ) . \frac{62}{(a+b)(b+c)(a+c)}.

The next realisation is that 3 ( a + b ) ( b + c ) ( a + c ) = ( a + b + c ) 3 ( a 3 + b 3 + c 3 ) = 7 3 ( a 3 + b 3 + c 3 ) . 3(a+b)(b+c)(a+c)=(a+b+c)^3-(a^3+b^3+c^3) = 7^3 -(a^3+b^3+c^3).

Combining these two equations, we find that a 3 + b 3 + c 3 = ( a + b + c ) 3 3 ( a + b ) ( b + c ) ( a + c ) = 7 3 3 62 31 = 7 3 6 = 337. a^3+b^3+c^3=(a+b+c)^3 - 3(a+b)(b+c)(a+c) = 7^3 - 3\cdot\frac{62}{31}=7^3-6 = 337.

Note that in fact, no such real numbers a , b , c a,b,c exist, so this problem is faulty. (check that a 2 a^2 , b 2 b^2 , and c 2 c^2 do not satisfy AM-GM)

Caleb Stanford - 7 years, 4 months ago

Log in to reply

Just see if one of the values is 6.96760404

kalyan pakala - 7 years, 3 months ago

AM-GM are for positive real numbers

Krishna Sharma - 6 years, 8 months ago

a , b , c is not itself satisfying AM-GM inequality , how come they be real numbers

Smit Anand - 7 years, 4 months ago

Log in to reply

AM-GM only applies to nonnegative real numbers, so you have to use a 2 , b 2 , a^2, b^2, and c 2 c^2 .

Caleb Stanford - 7 years, 4 months ago

Log in to reply

yup my fault and it is not satisfying a^2 , b^2, c^2 also

Smit Anand - 7 years, 4 months ago

After getting (a + b) (b + c) (c +a) = 2 , On expanding (a + b) (b + c) (c +a) we get :- ( a2 b + a2c + ab2 + 2abc + ac2 + b2c + bc2) = 2, which can be rewritten as:- ab(a + b) + bc(b +c) +ac(a + b) = 2. Now as a + b + c = 7; a + b = 7 – c, b +c = 7 – a and a + c = 7 – b. Putting these values on the above equation we get:- 7(ab + bc + ca) – abc = 2 Now since ab + bc + ca = 13 {from (a + b + c)2 - a2 +b2+c2} Therefore abc = 89. Now a3 + b3 + c3 – 3abc = (a + b + c)( a2 +b2+c2 -ab - bc - ca) So , a3 + b3 + c3 - 3abc = 7(23 – 10) = 70 a3 + b3 + c3 = 70 + 3(89) = 70 + 267 = 337

Pushpak Roy - 7 years, 3 months ago

Not real number solution.

Marko Hojan - 7 years, 3 months ago

i was closed, but failed...

raj sourav - 7 years ago

Excellent thanks

محمد محمد أمين - 7 years, 4 months ago

Thanks for such a nice explanation....

RAKESH PRADHAN - 7 years, 4 months ago
Sagnik Saha
Jan 28, 2014

1 a + b + 1 b + c + 1 c + a = 31 \dfrac{1}{a+b} + \dfrac{1}{b+c} + \dfrac{1}{c+a} = 31 . Simplification leads to

a 2 + b 2 + c 2 + 3 a b + 3 b c + 3 a c ( a + b ) ( b + c ) ( c + a ) = 31 \dfrac{a^2 + b^2+c^2+3ab+3bc+3ac}{(a+b)(b+c)(c+a)} = 31 .

Now, a b + b c + a c = ( a + b + c ) 2 ( a 2 + b 2 + c 2 ) 2 = 13 ab + bc+ac=\dfrac{(a+b+c)^2 -(a^2 + b^2 + c^2)}{2} = 13 . So we have,

23 + 39 ( a + b ) ( b + c ) ( c + a ) = 31 \dfrac{23+39}{(a+b)(b+c)(c+a)} = 31 \implies ( a + b ) ( b + c ) ( c + a ) = 2 (a+b)(b+c)(c+a) = 2

Finally a 3 + b 3 + c 3 = ( a + b + c ) 3 3 ( a + b ) ( b + c ) ( c + a ) = 7 3 3 × 2 = 343 6 = 337 \large{a^3+b^3+c^3 = (a+b+c)^3 - 3(a+b)(b+c)(c+a) = 7^3 - 3 \times 2 = 343 - 6 = \boxed{337}}

343-6=337

Abhishek Harlalka - 7 years, 4 months ago

wonderful

محمد محمد أمين - 7 years, 4 months ago

i did it exactly the same way......cheers!!!!

Aman Jaiswal - 7 years, 4 months ago

really impressive

Balendra Kumar - 7 years, 4 months ago

Same Way !!!

abc xyz - 4 years, 11 months ago

Excellent dude!!

anant pathak - 7 years, 4 months ago
Vikram Pandya
Jan 28, 2014

( a + b + c ) 2 = a 2 + b 2 + c 2 + 2 ( a b + b c + a c ) (a+b+c)^{2} = a^{2}+b^{2}+c^{2}+2*(ab+bc+ac) Hence 7 2 = 49 = 23 + 2 ( a b + b c + a c ) 7^{2} = 49 = 23 + 2*(ab+bc+ac) Therefore (ab+bc+ac) = 13 -------> (1)

( a + b + c ) 3 = a 3 + b 3 + c 3 + 3 ( a + b ) ( b + c ) ( c + a ) \[ H e n c e \[ 7 3 = 343 = a 3 + b 3 + c 3 + 3 ( a + b ) ( b + c ) ( c + a ) (a+b+c)^{3} = a^{3}+b^{3}+c^{3}+3*(a+b)(b+c)(c+a)\[ Hence \[7^{3} = 343 = a^{3}+b^{3}+c^{3} +3*(a+b)(b+c)(c+a) Therefore a 3 + b 3 + c 3 = 343 3 ( a + b ) ( b + c ) ( c + a ) a^{3}+b^{3}+c^{3} = 343 - 3*(a+b)(b+c)(c+a) -----------> (2)

We are given that 1 a + b + 1 b + c + 1 c + a = 31 \frac{1}{a+b}+\frac{1}{b+c}+\frac{1}{c+a} = 31 which is on simplification a 2 + b 2 + c 2 + 3 ( a b + b c + a c ) ( a + b ) ( b + c ) ( c + a ) = 31 \frac{a^{2}+b^{2}+c^{2}+3*(ab+bc+ac)}{(a+b)(b+c)*(c+a)} = 31 Which is 23 + 3 13 ( a + b ) ( b + c ) ( c + a ) = 31 \frac{23+3*13}{(a+b)(b+c)(c+a)} = 31 Hence (a+b)(b+c)*(c+a) = 62/31 = 2 -------------------> (3)

Substituting (3) in (2) we get a 3 + b 3 + c 3 = 343 3 2 = 337 a^{3}+b^{3}+c^{3} = 343 - 3*2 = \boxed{337}

you are excellent

محمد محمد أمين - 7 years, 4 months ago

Excellent solution.That's great.

Punsisi Bandara Kiridana - 7 years, 4 months ago

you, sir, rock!

Yash Bhagwat - 7 years, 4 months ago

Superb solution......!!!!!!!

Rohit Nair - 7 years, 3 months ago
Fikri Halim
Jan 28, 2014

We'll use this identity to find the value of a 3 + b 3 + c 3 a^3 + b^3 + c^3 : ( a + b + c ) 3 = a 3 + b 3 + c 3 + 3 ( a + b + c ) ( a b + b c + a c ) 3 ( a b c ) (a+b+c)^3=a^3+b^3+c^3 +3(a+b+c)(ab+bc+ac)-3(abc)

From 1st and 2nd equation in the problem, we can find a b + b c + a c ab+bc+ac ( a + b + c ) 2 = a 2 + b 2 + c 2 + 2 ( a b + b c + a c ) (a+b+c)^2 = a^2+b^2+c^2+2(ab+bc+ac)

7 2 = 23 + 2 ( a b + b c + a c ) 7^2= 23 + 2(ab+bc+ac)

a b + b c + a c = 13 ab+bc+ac= 13

Meanwhile, we can write the 3rd equation into this form : 1 7 a + 1 7 b + 1 7 c = 31 \frac{1}{7-a}+\frac{1}{7-b}+\frac{1}{7-c}=31 .

Then, ( 7 a ) ( 7 b ) + ( 7 b ) ( 7 c ) + ( 7 c ) ( 7 a ) ( 7 a ) ( 7 b ) ( 7 c ) = 31 \frac{(7-a)(7-b)+(7-b)(7-c)+(7-c)(7-a)}{(7-a)(7-b)(7-c)}=31

( 7 a ) ( 7 b ) + ( 7 b ) ( 7 c ) + ( 7 c ) ( 7 a ) = 31 ( 7 a ) ( 7 b ) ( 7 c ) (7-a)(7-b)+(7-b)(7-c)+(7-c)(7-a)=31(7-a)(7-b)(7-c)

3. 7 2 14 ( a + b + c ) ( a b + b c + a c ) = 31 ( 7 3 7 2 ( a + b + c ) + 7 ( a b + b c + a c ) a b c ) 3.7^2 -14(a+b+c)-(ab+bc+ac) = 31(7^3-7^2(a+b+c)+7(ab+bc+ac)-abc)

3. 7 2 14 ( 7 ) ( 13 ) = 31 ( 7 3 7 3 + 7 ( 13 ) a b c ) 3.7^2 - 14(7) - (13) = 31 (7^3-7^3+7(13)-abc)

a b c = 89 abc =89

Substitute a b + b c + a c = 13 , a b c = 89 ab+bc+ac= 13, abc =89 and a + b + c = 7 a+b+c=7 into ( a + b + c ) 3 = a 3 + b 3 + c 3 + 3 ( a + b + c ) ( a b + b c + a c ) 3 ( a b c ) (a+b+c)^3=a^3+b^3+c^3 +3(a+b+c)(ab+bc+ac)-3(abc) .

Then, ( a + b + c ) 3 = a 3 + b 3 + c 3 + 3 ( a + b + c ) ( a b + b c + a c ) 3 ( a b c ) (a+b+c)^3=a^3+b^3+c^3 +3(a+b+c)(ab+bc+ac)-3(abc)

( 7 ) 3 = a 3 + b 3 + c 3 + 3 ( 7 ) ( 13 ) 3 ( 89 ) (7)^3=a^3+b^3+c^3 +3(7)(13)-3(89)

a 3 + b 3 + c 3 = 337 a^3+b^3+c^3 = \boxed{337}

Excellent

محمد محمد أمين - 7 years, 4 months ago

cool

Niranjan Khanderia - 7 years, 2 months ago
Caleb Stanford
Jan 28, 2014

I have computed the answer to be 337 337 ; however, the truth is that no such real numbers exist!!

Using the third equation 1 a + b + 1 b + c + 1 a + c = 31 \frac{1}{a + b} + \frac{1}{b + c} + \frac{1}{a + c} = 31 , we can show that a b c = 89 abc = 89 (Detailed proof at the end of this post.) Then you will recall that the AM-GM inequality holds for all nonnegative real numbers, so in particular consider a 2 , b 2 , a^2, b^2, and c 2 c^2 . We have that a 2 + b 2 + c 2 3 a 2 b 2 c 2 3 23 3 ( a b c ) 2 3 23 3 8 9 2 3 2 3 3 3 3 8 9 2 12167 27 7921 12167 7921 27 = 213867 \begin{aligned} \frac{a^2 + b^2 + c^2}{3} &\ge \sqrt[3]{a^2 b^2 c^2} \\ \frac{23}{3} &\ge \sqrt[3]{(abc)^2} \\ \frac{23}{3} &\ge \sqrt[3]{89^2} \\ \frac{23^3}{3^3} &\ge 89^2 \\ \frac{12167}{27} &\ge 7921 \\ 12167 &\ge 7921 \cdot 27 = 213867 \end{aligned} which is a contradiction, showing that a , b , c a,b,c do not, in fact, exist.

Another way to see that no real numbers exist is to compute the polynomial ( x a ) ( x b ) ( x c ) = x 3 ( a + b + c ) x 2 + ( a b + b c + c a ) x a b c (x - a)(x - b)(x - c) = x^3 - (a + b + c)x^2 + (ab + bc + ca)x - abc . It is straightforward to show that a b + b c + c a = 13 ab + bc + ca = 13 (this is at the end of this post also), so the polynomial is x 3 7 x 2 + 13 x 89 x^3 - 7x^2 + 13x - 89 which one can check has no real roots, either by calculating that the discriminant is negative , or by graphing the equation .

Over the complex numbers these sorts of problems are guaranteed a solution, but over the real numbers they are not. One should be careful when formulating such a problem to make sure there is actually a solution!


Here are proofs of the basic facts used above.

First, a b + b c + c a = ( a + b + c ) 2 ( a 2 + b 2 + c 2 ) 2 = 7 2 23 2 = 13 ab + bc + ca = \frac{(a + b + c)^2 - (a^2 + b^2 + c^2)}{2} = \frac{7^2 - 23}{2} = 13 Second, 1 a + b + 1 b + c + 1 c + a = 31 ( a + c ) ( b + c ) + ( a + b ) ( a + c ) + ( a + b ) ( b + c ) = 31 ( a + b ) ( b + c ) ( c + a ) \begin{aligned} \frac{1}{a + b} + \frac{1}{b + c} + \frac{1}{c + a} &= 31 \\ \implies (a + c)(b + c) + (a + b)(a + c) + (a + b)(b + c) &= 31(a + b)(b + c)(c + a) \\ \end{aligned} So we have a 2 + b 2 + c 2 + 3 ( a b + b c + c a ) = 31 [ a 2 b + a 2 c + b 2 a + b 2 c + c 2 a + c 2 b + 2 a b c ] a 2 + b 2 + c 2 + 3 ( a b + b c + c a ) = 31 [ ( a + b + c ) ( a b + b c + c a ) a b c ] a 2 + b 2 + c 2 + 3 ( a b + b c + c a ) = 31 ( a + b + c ) ( a b + b c + c a ) 31 a b c \begin{aligned} a^2 + b^2 + c^2 + 3(ab + bc + ca) &= 31[a^2b + a^2c + b^2a + b^2c + c^2a + c^2b + 2abc] \\ a^2 + b^2 + c^2 + 3(ab + bc + ca) &= 31[(a + b + c)(ab + bc + ca) - abc] \\ a^2 + b^2 + c^2 + 3(ab + bc + ca) &= 31(a + b + c)(ab + bc + ca) - 31abc \end{aligned} Therefore, 31 a b c = 31 ( a + b + c ) ( a b + b c + c a ) 3 ( a b + b c + c a ) ( a 2 + b 2 + c 2 ) = 31 ( 7 ) ( 13 ) 3 ( 13 ) 23 = 31 ( 91 ) 39 23 = 31 ( 91 ) 31 ( 2 ) a b c = 91 2 = 89. \begin{aligned} 31abc &= 31(a + b + c)(ab + bc + ca) - 3(ab + bc + ca) - (a^2 + b^2 + c^2) \\ &= 31(7)(13) - 3(13) - 23 = 31(91) - 39 - 23 = 31(91) - 31(2) \\ \implies abc &= 91 - 2 = 89. \end{aligned}

Thanks for the observation! I was debating when I wrote this problem whether it would be appropriate to say that they are real values, but I wasn't sure how to prove it. I guess it would have been better to say that they are complex numbers. :)

Brian Yao - 7 years, 4 months ago

Log in to reply

You're welcome. However, do you realize that this renders your problem explicitly invalid? The correct answer to the problem is "no solutions", which isn't an option. I'm not just making an "observation", I'm explaining why your problem is wrong. Please fix it if at all possible. Thank you!

Caleb Stanford - 7 years, 4 months ago

Log in to reply

Yes, of course, I do realize the problem is faulty. However, the problem isn't asking for the individual values of a a , b b , and c c , but an expression involving these values. The main focus of this problem is algebraic manipulation, so the subtlety of the values being real shouldn't have any impact. In addition, I cannot change the title of the problem (as far as I know), so having two contradicting statements in one problem would be silly. Once I have the option to change the title as well, I will fix this.

Brian Yao - 7 years, 4 months ago
Brian Yao
Jan 21, 2014

First, note the following general equations for values a a , b b , and c c :

  1. ( a + b + c ) 2 = a 2 + b 2 + c 2 + 2 ( a b + a c + b c ) (a+b+c)^2=a^2+b^2+c^2+2(ab+ac+bc)
  2. ( a + b + c ) 3 = a 3 + b 3 + c 3 + 3 ( a 2 ( b + c ) + b 2 ( a + c ) + c 2 ( a + b ) + 2 a b c ) (a+b+c)^3=a^3+b^3+c^3+3(a^2(b+c)+b^2(a+c)+c^2(a+b)+2abc)
  3. 1 a + b + 1 b + c + 1 a + c = a 2 + b 2 + c 2 + 3 ( a b + a c + b c ) ( a + b ) ( b + c ) ( a + c ) \frac{1}{a+b}+\frac{1}{b+c}+\frac{1}{a+c}=\frac{a^2+b^2+c^2+3(ab+ac+bc)}{(a+b)(b+c)(a+c)}
  4. ( a + b ) ( b + c ) ( a + c ) = a 2 ( b + c ) + b 2 ( a + c ) + c 2 ( a + b ) + 2 a b c (a+b)(b+c)(a+c)=a^2(b+c)+b^2(a+c)+c^2(a+b)+2abc

Here, we can see that:

  • Using equation 1 1 , the value of a b + a c + b c ab+ac+bc is ( a + b + c ) 2 ( a 2 + b 2 + c 2 ) 2 = 49 23 2 = 13 \frac{(a+b+c)^2-(a^2+b^2+c^2)}{2}=\frac{49-23}{2}=13 .
  • Letting the right hand side of equation 3 3 be equal to the given value of 31 31 and multiplying the denominator over, we get that a 2 + b 2 + c 2 + 3 ( a b + a c + b c ) = 23 + 3 ( 13 ) = 62 = 31 ( a + b ) ( b + c ) ( a + c ) a^2+b^2+c^2+3(ab+ac+bc)=23+3(13)=62=31(a+b)(b+c)(a+c) ( a + b ) ( b + c ) ( a + c ) = 2 \Rightarrow (a+b)(b+c)(a+c)=2 .
  • Note that, using equation 4 4 , we can make a substitution and rewrite equation 2 2 : ( a + b + c ) 3 = a 3 + b 3 + c 3 + 3 ( a + b ) ( b + c ) ( a + c ) (a+b+c)^3=a^3+b^3+c^3+3(a+b)(b+c)(a+c) . Thus, solving for a 3 + b 3 + c 3 a^3+b^3+c^3 yields a 3 + b 3 + c 3 = ( a + b + c ) 3 3 ( a + b ) ( b + c ) ( a + c ) = 343 3 ( 2 ) = 337 a^3+b^3+c^3=(a+b+c)^3-3(a+b)(b+c)(a+c)=343-3(2)=\boxed{337} .

Excellent

محمد محمد أمين - 7 years, 4 months ago
Arpit Sah
Jan 30, 2014

a + b + c = 7

a 2 + b 2 + c 2 a^2 + b^2 + c^2 = 23

( a + b + c ) 2 (a + b + c)^2 = a 2 + b 2 + c 2 a^2 + b^2 + c^2 + 2( ab + bc + ca )

7 2 7^2 = 23 + 2( ab + bc + ca )

49 = 23 + 2( ab + bc + ca )

2( ab + bc + ca ) = 49 - 23

( ab + bc + ca ) = 13

( ab + bc + ca )( a + b + c ) = 91

(a^2b + a^2c + b^2a + b^2c + c^2a + c^2b) + 3abc = 91

Simplify : 1 a + b + 1 b + c + 1 a + c \frac{1}{a+b} + \frac{1}{b+c} + \frac{1}{a+c} .

1 a + b + 1 b + c + 1 a + c \frac{1}{a+b} + \frac{1}{b+c} + \frac{1}{a+c} = 3 ( a b + b c + a c ) + ( a 2 + b 2 + c 2 ) 2 a b c + ( a 2 b + a 2 c + b 2 a + b 2 c + c 2 a + c 2 b ) \frac{3(ab + bc + ac) + (a^2 + b^2 + c^2)}{2abc + (a^2b + a^2c + b^2a + b^2c + c^2a + c^2b)}

1 a + b + 1 b + c + 1 a + c \frac{1}{a+b} + \frac{1}{b+c} + \frac{1}{a+c} = 31

3 ( a b + b c + a c ) + ( a 2 + b 2 + c 2 ) 2 a b c + ( a 2 b + a 2 c + b 2 a + b 2 c + c 2 a + c 2 b ) \frac{3(ab + bc + ac) + (a^2 + b^2 + c^2)}{2abc + (a^2b + a^2c + b^2a + b^2c + c^2a + c^2b)} = 31

31 = 3 ( a b + b c + a c ) + ( a 2 + b 2 + c 2 ) 3 a b c + ( a 2 b + a 2 c + b 2 a + b 2 c + c 2 a + c 2 b ) a b c \frac{3(ab + bc + ac) + (a^2 + b^2 + c^2)}{3abc + (a^2b + a^2c + b^2a + b^2c + c^2a + c^2b) - abc }

[adding and subtracting 'abc' to the denominator]

31 = 3 ( 13 ) + ( 23 ) 3 a b c + ( a 2 b + a 2 c + b 2 a + b 2 c + c 2 a + c 2 b ) a b c \frac{3(13) + (23)}{3abc + (a^2b + a^2c + b^2a + b^2c + c^2a + c^2b) - abc }

39 + 23 91 a b c \frac{39 + 23}{91 - abc } = 31

62 91 a b c \frac{62}{91 - abc } = 31

91 - abc = 2

abc = 89

a 3 + b 3 + c 3 a^3 + b^3 + c^3 - 3abc = (a + b + c)[ a 2 + b 2 + c 2 a^2 + b^2 + c^2 - (ab + bc + ac)]

a 3 + b 3 + c 3 a^3 + b^3 + c^3 - 3(89) = 7(23 - 13)

a 3 + b 3 + c 3 a^3 + b^3 + c^3 - 267 = 7(10)

a 3 + b 3 + c 3 a^3 + b^3 + c^3 = 267 + 70

a 3 + b 3 + c 3 a^3 + b^3 + c^3 = 337

Charan Eswar
Feb 8, 2014

(a + b + c)3= a^3 + b^3+ c^3 + 3 [(a+b)(b+c)(a+c)] ----1

given

31 = [a^2+b^2+c^2 + 3(ab+bc+ac)] / [(a+b)(b+c)(a+c)]

== > [ (a+b)(b+c)(a+c)] = [a^2+b^2+c^2 + {3/2[(a + b + c)^2 - a^2+b^2+c^2]}] / 31

                                       =   [23 + { 3/2  [(7)^2 - 23]}]  / 31

                                       = [23 + { 3/2 [26] } ]  /  31

                                         =[23+ 39 ]   /  31

                                          =62 / 31  = 2 =    [ (a+b)(b+c)(a+c)]------2

                            substitute 2 in 1

                     7^3  =    a^3 + b^3+ c^3+  3(2)
                   343 - 6  =  a^3 + b^3+ c^3
                 ans:-  a^3 + b^3+ c^3  =  337
Ayush Shukla
Jan 29, 2014

(a+b+c)^3=a3+b3+c3+3(a2b+ab2+b2c+bc2+a2c+ac2)+6abc ab+bc+ca=((a+b+c)^2-(a2+b2+c2))/2=13 (a+b)(b+c)(c+a)=a2b+ab2+b2c+bc2+a2c+ac2+2abc=2 a2b+ab2+b2c+bc2+a2c+ac2=2-2abc now put all these values in first eqn. (a+b+c)^3=a3+b3+c3+3(a2b+ab2+b2c+bc2+a2c+ac2)+6abc 343 = a3+b3+c3+3(2-2abc)+6abc 343-6=a3+b3+c3 therefore a3+b3+c3=337

Bharat Karmarkar
Jan 28, 2014

Step1: get a^{2}b + b^{2}c + c^{2}a + ab^{2} + bc^{2} + ac^{2} + 2abc = 2 Step 2: Expand (a + b + c)^{3} and get the answer

sorry, I tried, but I do not understand how to type it properly.

Bharat Karmarkar - 7 years, 4 months ago

Log in to reply

See here

Rahul Saha - 7 years, 4 months ago
Kautsya Kanu
Jan 28, 2014

from (a+b+c)^{2} = a^{2}+b^{2}+c^{2}+2 (ab+bc+ca) , find ab+bc+ca; then from 3rd equation, by cross multiplication find (2 abc+a^{2}b+a b^{2}+....); then use (a+b+c)^{3} = a^{3}+b^{3}+c^{3}+3 (2 abc+a^{2}b+a b^{2}+....) to find answer....

Anish Kelkar
Jul 21, 2015

These 3 equations can be reduced into linear equation with two variables such that 1) y=3x+70 2) y=2x+159 . where y= a 3 \sum { a^{ 3 } } and x = a b c abc .

Souryajit Roy
Jul 12, 2014

2 ( a b + b c + c a ) = ( a + b + c ) 2 ( a 2 + b 2 + c 2 ) = 49 23 = 26 2(ab+bc+ca)=(a+b+c)^{2}-(a^{2}+b^{2}+c^{2})=49-23=26

or, a b + b c + c a = 13 ab+bc+ca=13

From the last equation, c y c ( a + b ) ( b + c ) = 31 ( a + b ) ( b + c ) ( c + a ) \sum_{cyc}(a+b)(b+c)=31(a+b)(b+c)(c+a)

or, c y c ( a b + a c + b 2 + b c ) = 31 ( a + b ) ( b + c ) ( c + a ) \sum_{cyc}(ab+ac+b^{2}+bc)=31(a+b)(b+c)(c+a)

or, c y c ( a b + b c + c a ) + c y c b 2 = 31 ( a + b ) ( b + c ) ( c + a ) \sum_{cyc}(ab+bc+ca)+\sum_{cyc}b^{2}=31(a+b)(b+c)(c+a)

or, 31 ( a + b ) ( b + c ) ( c + a ) = 3.13 + 23 = 62 31(a+b)(b+c)(c+a)=3.13+23=62 .Hence, ( a + b ) ( b + c ) ( c + a ) = 2 (a+b)(b+c)(c+a)=2

Now, ( a + b ) ( b + c ) ( c + a ) = ( a + b + c ) ( a b + b c + c a ) a b c = 91 a b c (a+b)(b+c)(c+a)=(a+b+c)(ab+bc+ca)-abc=91-abc .

Hence, a b c = 89 abc=89

Now, a , b , c a,b,c are the roots of x 3 7 x 2 + 13 x 89 = 0 x^{3}-7x^{2}+13x-89=0

Finally, i use the result that if a , b , c a,b,c are the roots of x 3 + p 1 x 2 + p 2 x + p 3 = 0 x^{3}+p_{1}x^{2}+p_{2}x+p_{3}=0 ,then a 3 , b 3 , c 3 a^{3},b^{3},c^{3} are the roots of x 3 ( p 1 3 3 p 1 p 2 + 3 p 3 ) x 2 x^{3}-(p_{1}^{3}-3p_{1}p_{2}+3p_{3})x^{2} + ( p 2 3 3 p 1 p 2 p 3 + 3 p 3 2 ) x +(p_{2}^{3}-3p_{1}p_{2}p_{3}+3p_{3}^{2})x + + p 3 3 = 0 p_{3}^{3}=0

Here, we just have to compute the second co-efficient.Hence the value of a 3 + b 3 + c 3 = ( ( 7 ) 3 3 ( 7 ) ( 13 ) + 3 ( 89 ) ) = 337 a^{3}+b^{3}+c^{3}=-((-7)^{3}-3(-7)(13)+3(-89))=337

Hùng Minh
Feb 11, 2014

a + b + c = 7 < = > ( a + b + c ) 2 = 49 = a 2 + b 2 + c 2 + 2 a b + 2 b c + 2 a c a+b+c=7 <=> (a+b+c)^{2} = 49 = a^{2}+b^{2}+c^{2}+2ab+2bc+2ac

< = > a b + b c + a c = 1 2 ( 49 ( a 2 + b 2 + c 2 ) ) = 1 2 ( 49 23 ) = 13 <=>ab+bc+ac = \frac{1}{2}(49-(a^{2}+b^{2}+c^{2}))=\frac{1}{2}(49-23)=13

1 a + b + 1 b + c + 1 a + c = ( a + c ) ( b + c ) a + b + ( a + b ) ( a + c ) b + c + ( a + b ) ( b + c ) a + c \frac{1}{a+b}+\frac{1}{b+c}+\frac{1}{a+c}=\frac{(a+c)(b+c)}{a+b}+\frac{(a+b)(a+c)}{b+c}+\frac{(a+b)(b+c)}{a+c}

= a 2 + b 2 + c 2 + 3 a b + 3 b c + 3 a c ( a + b ) ( b + c ) ( a + c ) = ( a + b + c ) 2 + a b + b c + a c ( a + b ) ( b + c ) ( a + c ) = \frac{a^{2}+b^{2}+c^{2}+3ab+3bc+3ac}{(a+b)(b+c)(a+c)}=\frac{(a+b+c)^2+ab+bc+ac}{(a+b)(b+c)(a+c)}

< = > ( a + b ) ( b + c ) ( a + c ) = ( a + b + c ) 2 + a b + b c + a c 31 = 49 + 13 31 = 2 <=> (a+b)(b+c)(a+c) = \frac{(a+b+c)^2+ab+bc+ac}{31} = \frac{49+13}{31} = 2

( a + b + c ) 3 = a 3 + b 3 + c 3 + 3 ( a + b ) ( b + c ) ( a + c ) (a+b+c)^{3} = a^{3}+b^{3}+c^{3} + 3(a+b)(b+c)(a+c)

< = > a 3 + b 3 + c 3 = ( a + b + c ) 3 3 ( a + b ) ( b + c ) ( a + c ) = 7 3 3 2 = 337 <=> a^{3}+b^{3}+c^{3} = (a+b+c)^{3} - 3(a+b)(b+c)(a+c) = 7^{3} - 3*2 = 337

(a+b+c)^3=a^3+b^3+c^3+3(a+b)(b+c)(c+a).............1 from the last equation we get 31(a+b)(b+c)(c+a)=(b+c)(c+a)+(a+b)(b+c)+(a+b)(c+a) =a^2+b^2+c^2+3(ab+bc+ca).................2 from the given first equation squaring both sides we get (a+b+c)^2=a^2+b^2+c^2+2(ab+bc+ca) 49=23+2(ab+bc+ca) ab=bc+ca=13 substitute in ....2 we get 31(a+b)(b+c)(c+a)=23+29=62 (a+b)(b+c)(c+a)=2 substitute in ....1 we get a^3+b^3+c^3=343-6=337

Victor Loh
Jan 31, 2014

a 3 + b 3 + c 3 3 a b c = ( a + b + c ) ( a 2 + b 2 + c 2 a b b c a c ) a^{3}+b^{3}+c^{3}-3abc = (a+b+c)(a^{2}+b^{2}+c^{2}-ab-bc-ac) a 3 + b 3 + c 3 = ( a + b + c ) ( a 2 + b 2 + c 2 a b b c a c ) + 3 a b c a^{3}+b^{3}+c^{3} = (a+b+c)(a^{2}+b^{2}+c^{2}-ab-bc-ac)+3abc a 3 + b 3 + c 3 = 7 ( 23 a b b c a c ) + 3 a b c a^{3}+b^{3}+c^{3} = 7(23-ab-bc-ac)+3abc 1 a + b + 1 b + c + 1 a + c = 31 \frac{1}{a+b}+\frac{1}{b+c}+\frac{1}{a+c} = 31 ( b + c ) ( a + c ) + ( a + b ) ( a + c ) + ( a + b ) ( b + c ) ( a + b ) ( b + c ) ( a + c ) = 31 \frac{(b+c)(a+c)+(a+b)(a+c)+(a+b)(b+c)}{(a+b)(b+c)(a+c)} = 31 a b + b c + a c + c 2 + a 2 + a c + a b + b c + a b + a c + b 2 + b c ( a + b ) ( b + c ) ( a + c ) = 31 \frac{ab+bc+ac+c^{2}+a^{2}+ac+ab+bc+ab+ac+b^{2}+bc}{(a+b)(b+c)(a+c)} = 31 a 2 + b 2 + c 2 + 3 ( a b + b c + a c ) ( a + b ) ( b + c ) ( a + c ) = 31 \frac{a^{2}+b^{2}+c^{2}+3(ab+bc+ac)}{(a+b)(b+c)(a+c)} = 31 23 + 3 ( a b + b c + a c ) ( a + b ) ( b + c ) ( a + c ) = 31 \frac{23+3(ab+bc+ac)}{(a+b)(b+c)(a+c)} = 31 a + b = 7 c a+b = 7-c a + c = 7 b a+c = 7-b b + c = 7 a b+c = 7-a ( b + c ) ( a + c ) + ( a + b ) ( a + c ) + ( a + b ) ( b + c ) ( a + b ) ( b + c ) ( a + c ) = 31 \frac{(b+c)(a+c)+(a+b)(a+c)+(a+b)(b+c)}{(a+b)(b+c)(a+c)} = 31 ( 7 a ) ( 7 b ) + ( 7 c ) ( 7 b ) + ( 7 c ) ( 7 a ) ( a + b ) ( b + c ) ( a + c ) = 31 \frac{(7-a)(7-b)+(7-c)(7-b)+(7-c)(7-a)}{(a+b)(b+c)(a+c)} = 31 49 7 b 7 a + a b + 49 7 b 7 c + b c + 49 7 a 7 c + a c ( a + b ) ( b + c ) ( a + c ) = 31 \frac{49-7b-7a+ab+49-7b-7c+bc+49-7a-7c+ac}{(a+b)(b+c)(a+c)} = 31 147 14 a 14 b 14 c + a b + b c + a c ( a + b ) ( b + c ) ( a + c ) = 31 \frac{147-14a-14b-14c+ab+bc+ac}{(a+b)(b+c)(a+c)} = 31 147 14 ( a + b + c ) + a b + b c + a c ( a + b ) ( b + c ) ( a + c ) = 31 \frac{147-14(a+b+c)+ab+bc+ac}{(a+b)(b+c)(a+c)} = 31 147 14 ( 7 ) + a b + b c + a c ( a + b ) ( b + c ) ( a + c ) = 31 \frac{147-14(7)+ab+bc+ac}{(a+b)(b+c)(a+c)} = 31 147 98 + a b + b c + a c ( a + b ) ( b + c ) ( a + c ) = 31 \frac{147-98+ab+bc+ac}{(a+b)(b+c)(a+c)} = 31 49 + a b + b c + a c ( a + b ) ( b + c ) ( a + c ) = 31 \frac{49+ab+bc+ac}{(a+b)(b+c)(a+c)} = 31 23 + 3 ( a b + b c + a c ) = 49 + a b + b c + a c 23+3(ab+bc+ac) = 49+ab+bc+ac 2 ( a b + b c + a c ) = 49 23 = 26 2(ab+bc+ac) = 49-23 = 26 a b + b c + a c = 13 ab+bc+ac = 13 49 + 13 = 62 49+13 = 62 ( a + b ) ( b + c ) ( a + c ) = 2 (a+b)(b+c)(a+c)=2 a 2 ( b + c ) + b 2 ( a + c ) + c 2 ( a + b ) + 2 a b c = 2 a^{2}(b+c)+b^{2}(a+c)+c^{2}(a+b)+2abc=2 a 2 ( 7 a ) + b 2 ( 7 b ) + c 2 ( 7 c ) + 2 a b c = 2 a^{2}(7-a)+b^{2}(7-b)+c^{2}(7-c)+2abc=2 7 a 2 a 3 + 7 b 2 b 3 + 7 c 2 c 3 + 2 a b c = 2 7a^{2}-a^{3}+7b^{2}-b^{3}+7c^{2}-c^{3}+2abc=2 7 ( a 2 + b 2 + c 2 ) 7 ( 23 a b b c a c ) 3 a b c + 2 a b c = 2 7(a^{2}+b^{2}+c^{2})-7(23-ab-bc-ac)-3abc+2abc=2 7 ( 23 ) 7 ( 10 ) a b c = 2 7(23)-7(10)-abc=2 161 70 2 = a b c 161-70-2=abc a b c = 89 abc=89 3 a b c = 267 3abc=267 267 + 70 = 337 267+70=\boxed{337}

My attempt to write a long solution without words XD

Victor Loh - 7 years, 4 months ago
Anish Das
Jan 29, 2014

a^3 +b^3+c^3=3abc+(a+b+c)(a^2 + b^2 + c^2 -ab - bc - ca)

  1. (a+b+c)^2 =a ^ 2 +b ^ 2 + c ^ 2 +2*(ab +bc+ca)

solving 1 we get ab+bc+ca=13

  1. 1/(b+c) +1/(c+a) + 1/(b+a)=31

== a^2+b^2+c^2 +3(ab+bc+ca)/(a+b)(b+c)(c+a)=31

or (a+b)(b+c)(c+a)=2

or ba^2 + ab^2 +ca^2 +ac^2 + bc^2 + cb^2 + 2abc=2

adding abc both side

we get lhs=rhs

(a+b+c)(bc+ca+ab)=2+abc

or abc=89

put all the values to get a^3 + b^3 +c^3 = 337

From the 1st and 2nd equations we get ab + bc + ca = 13; Plug the values of a + b+ c, ab + bc + ca in the 3rd equation to obtain abc = 89. So a, b, c are the roots of the equation x^3 -7x^2 + 13x - 89 = 0. Hence a^3 + b^3 + c^3 = 7(a^2 + b^2 +c^2) - 13(a + b + c) +3(89) = 337

0 pending reports

×

Problem Loading...

Note Loading...

Set Loading...